Summen in die vorgegebene Form umschreiben

Aufrufe: 1250     Aktiv: 31.10.2019 um 21:14

0

Hallo, ich soll die Summe: \(\sum_{k=1}^n k\) in die folgende Form umschreiben: \(\sum_{k=1}^n (n+1-k)\). Ich habe keine Ahnnung, wie man vorgehen soll. 

 

Diese Frage melden
gefragt

Student, Punkte: 370

 

Schreib dir Mal diese Summen für n=4 oder n=5 aus ;)   ─   jojoliese 26.10.2019 um 12:01

Habe ich gemacht, was bringt das? Für das Erste habe ich "1+2+3+4=10" und für das zweite "4+3+2+1=10". Das geht gleichsam von der höchsten Zahl los. Und nun? Also nochmal: Bei der gegebenen Form wird k=? und n=?gesucht.   ─   kamil 27.10.2019 um 19:46
Kommentar schreiben
2 Antworten
0

Hallo,

du kannst die Gleichheit auch anders zeigen:

$$\sum_{k=1}^nk-\sum_{k=1}^n(n+1-k)=\sum_{k=1}^n(k-n-1+k)=\sum_{k=1}^n2k-(n+1)\sum_{k=1}^n1$$

$$=2\sum_{k=1}^nk-(n+1)n=2\frac{(n+1)n}{2}-(n+1)n=(n+1)n-(n+1)n=0$$

Und da die Differenz von beiden \(0\) sind, müssen sie gleich sein. Wenn du Summen über Konstanten oder die Gaußsche Summenformel nicht kennst, empfehle ich dir die Videos:

(Analysis 056 - Gaußsche Summenformel (mit Beweis))

(Analysis 049 - Summen- und Produktzeichen)

Diese Antwort melden
geantwortet

Student, Punkte: 2.6K

 

Hallo, vielen dank für die Videos. Ich konnte alles nachvollziehen, außer die Stelle mit den abgespaltenen Summanden. Ich konnte nicht nachvollziehen, welcher es war. Und hier bis zu 2 Zeile alles verstanden. Ab da keine Ahnung, von wo das "n" hinter dem Klammer, also ...-(n+1)n kommt und das was danach kommt xD. Die Antworten davor habe ich jetzt 100% verstanden. Vielen dank!!! :)   ─   kamil 31.10.2019 um 17:29

Es freut mich, dass dir die Videos geholfen haben! Hier passiert folgendes in der zweiten Zeile:
Es gilt \(\sum_{k=1}^n1=n\), weil man über die \(1\) summiert. Daher kommt das \(n\) hinter dem \((n+1)\) und in der zweiten Zeile ist der erste Schritt das Verwenden der Gaußschen Summenformel und danach vereinfacht! :)
  ─   endlich verständlich 31.10.2019 um 18:01

Kommentar schreiben

0

Hallo,

worauf jojoliese hinaus wollte, ist das die Reihenfolge der Summanden umgekehrt wird durch die neue Vorschrift. 

Grüße Christian

Diese Antwort melden
geantwortet

Sonstiger Berufsstatus, Punkte: 29.81K

 

Hallo, das habe ich verstanden. Verstehe aber immer noch nicht, was "k" und "n" ist. Würde mich freuen, wenn der Weg erläutert werden könnte.

Liebe Grüße,
Kamil
  ─   kamil 28.10.2019 um 18:30

Die Reihe durchläuft alle natürlichen Zahlen ab einem bestimmten Startwert (\(k=1\)) bis zu einem Endwert (\(n\)).
Nun wollen wir aber beim Wert \( n \) anfangen und jedes mal einen abziehen. Schreiben wir das mal ganz allgemein auf
$$ n + (n-1) + (n-2) + (n-3) + \ldots + 2 + 1 $$
Das wäre die folgende Summe
$$ \sum_{k=0}^{n-1} n- k $$
Diese fängt bei \(k=0 \) an, da wir im ersten Summanden nichts abziehen, wir wollen die Summe ja mit \( n \) beginnen.
Die Summe geht nur bis \( n-1 \), da
$$ n-(n-1) = n-n+1 = 1 $$
Nun machen wir noch eine Indexverschiebung und erhalten
$$ \sum_{k+1=0}^{(n+1)-1} n-(k-1) = \sum_{k=1}^n n-k+1 $$

Grüße Christian
  ─   christian_strack 28.10.2019 um 20:09

Christian hat sich verschrieben, er wollt die \(+1\) nicht zum \(k\) sondern zur \(0\) schreiben. Bei diesem Indexshift fängst du einfach 1 später an und hörst 1 später auf, damit aber das Gleiche da steht, muss du beim \(k\) das nicht im Index steht wieder eins abziehen! :)   ─   endlich verständlich 31.10.2019 um 14:58

Oh ja danke für die Korrektur :)

Grüße Christian
  ─   christian_strack 31.10.2019 um 15:08

Aber wieso zur "0"? Alles was man verändert, muss doch bei "k" stattfinden?
  ─   kamil 31.10.2019 um 15:10

Die 0 und das n-1 sind doch die Werte die k annimmt! :)   ─   endlich verständlich 31.10.2019 um 15:10

Vielen Dank für die Antworten! Jetzt sitzt das! :)   ─   kamil 31.10.2019 um 17:18

Kommentar schreiben